Unexpectedly rejected almost everywhere, Please explain..

  • This has become our largest and most active forum because the physics GRE is just one aspect of getting accepted into a graduate physics program.
  • There are applications, personal statements, letters of recommendation, visiting schools, anxiety of waiting for acceptances, deciding between schools, finding out where others are going, etc.

jigsaw
Posts: 40
Joined: Sun Mar 17, 2013 10:59 am

Unexpectedly rejected almost everywhere, Please explain..

Post by jigsaw » Sun Mar 17, 2013 1:56 pm

Quite unexpectedly I got rejected almost everywhere. Please take a look at my profile below, and share your opinion. Why do you think I got rejected virtually everywhere? What went so horribly wrong? Thanks a lot for your time.

Undergrad Institution: MIT
Major(s): Physics
Minor(s): None
GPA in Major: 4.7/5.0 (Please see explanation below)
Overall GPA: 4.4/5.0 (Please see explanation below)
Length of Degree: 4 years as usual, 2008-2012 (Please see explanation below)
Position in Class: Not sure, but probably slightly below average
Type of Student: International Male

GRE Scores :
Q: 170 (99%)
V: 154 (61%)
W: 4.0 (49%)
P: 990 (95%)

Research Experience: 4 UROPs at MIT LNS (hep-ex) and 1 UROP at MIT Kavli (astro) - none in hep-th though, No publication. (I didn't think that was a big disadvantage, I heard very few undergraduates have research experience and/or publication in hep-th)

Awards/Honors/Recognitions: some scholarships for 'scholastic performance', some direct funding & once honorary special funding for UROP - thats all

Pertinent Activities or Jobs: taught math for SAT to high school students in MIT ESP in my freshman year, graded introductory physics courses at MIT (mostly freshman classes, one sophomore class) pretty much every semester after my freshman year.

Any Miscellaneous Accomplishments that Might Help: I got into MIT from an extremely underprivileged background (and I had to overcome many obstacles), I don't know if that matters for graduate admission purposes, but I did explain it in my SOP.

Special Bonus Points: [1] 5 grad classes (QM I & II, Stat Mech I, QFT I, GR) and two special undergrad classes (Early Universe by Prof. Alan Guth, and String Theory : Undergraduate by Prof. Barton Zwiebach) - all A's. [2] 6 recommenders where the grad school allows it : 4 from hep-th, 1 from hep-ex, 1 from astro, 3/4 recommenders (all from hep-th) where the grad school doesn't allow more. [3] Not sure if this is an advantage or not, but my recommenders are pretty well known and most have done their Undergraduate/PhD/PostDoc at MIT, Caltech, Harvard, Stanford, Princeton, UC Berkeley.

Any Other Info That Shows Up On Your App and Might Matter: [1] I graduated in 2012, but I'm applying for graduate schools in US for 2013. Actually I applied for British Universities last year, and I got offers of admission from University of Cambridge (Part III of Math Tripos) and Imperial College London (MSc in QFFF) :D , but for unknown reasons all of my scholarship applications got rejected, and I received no funding whatsoever :cry: . So I had to take a year off in my home country and selfstudy using materials downloaded from MIT OCW & iTunes U (mostly Stanford, a little bit of Yale). I explained the situation in my SOP and also attached the offer letters. [2] I took 19 physics courses at MIT, and received A in 16 of them. The exceptions were : C in Quantum Physics I (shouldn't matter, I got A's in Quantum Physics II & III, as well as grad Quantum Mechanics I & II after all), B in Electromagnetism II (shouldn't matter either, I graded freshman Electromagnetism for 4 semesters to make up for any deficiency), and C in Experimental Physics I (I'm neither good nor interested in experiments, I tried really hard but apparently without much success; I don't think it would matter though, since I'm applying for hep-th, not hep-ex) - all in 2010 (calendar year), everything else are straight A's. [3] I failed to do any better than B's in Humanities & Chemistry, I even got C in Biology, and a mix of A's & B's in Math - these non physics courses were quite a few in number, which lowered my overall GPA significantly. I also had a bad start, and I got term GPA like 3.8/5.0 and 3.9/5.0 in the initial phase. I got hold of it, and received term GPA 4.8/5.0 in each of my last 3 semesters at MIT, but due to the bad start, my overall GPA didn't reach 4.5/5.0 as you can see above. [4] I had prolonged illness in my senior year, due to which I had to switch from the Focused Option of Physics Major to the Flexible Option (meaning no Senior Thesis) - I explained it in my SOP and also added a letter from MIT Medical confirming the health issue.

Research Interests: String Theory (main), Phenomenology (side), Inflation Cosmology (side)

Applying to Where:

Reach :

MIT - Physics - hep-th - Rejected
Caltech - Physics - hep-th - Rejected
Harvard - Physics - hep-th - Rejected
Stanford - Physics - hep-th - Rejected
Princeton - Physics - hep-th - Rejected
UC Berkeley - Physics - hep-th - Rejected
Chicago - Physics - hep-th - Rejected
Cornell - Physics - hep-th - Rejected

Match :

UCSB - Physics - hep-th - Rejected

Safety :

UMich - Physics - hep-th - Rejected
UT Austin - Physics - hep-th - Rejected
Stony Brook - Physics - hep-th - MA without support - declined
Rutgers - Physics - hep-th - Offer with TA support - accepted :|

Bonus :

Cambridge - DAMTP - hep-th - Part III without support - on 9/20 :evil:
Oxford - Physics - hep-th - Rejected

I know my profile isn't perfect (nor am I the next Witten :wink: ), but I didn't expect to get accepted at all 15 graduate schools I applied to, I honestly only expected to get accepted at 5-7 of those. What do you think caused this disaster? Feel free to share your opinion.
Last edited by jigsaw on Fri Sep 20, 2013 11:24 am, edited 14 times in total.

bfollinprm
Posts: 1203
Joined: Sat Nov 07, 2009 11:44 am

Re: Unexpectedly rejected almost everywhere, Please explain..

Post by bfollinprm » Sun Mar 17, 2013 3:27 pm

HEP-TH is very competitive, and you might have had a less than glowing review that put your application below others, almost all of which have very good stats. For a field as narrow as string theory, you're pigeonholing yourself to 1 or 2 profs at each university who may have no time for you, or preference for someone they're more familiar with.

There also isn't really a safety in that list, so I'm not overly surprised that this could have happened.

Ivan Fyodorovich
Posts: 32
Joined: Sat Feb 09, 2013 10:44 pm

Re: Unexpectedly rejected almost everywhere, Please explain..

Post by Ivan Fyodorovich » Sun Mar 17, 2013 3:31 pm

From other results, it looks like Stony Brook might be very low on funding this year as well, since quite a few applicants were accepted for the Master's program. I met one person at another open house who said that he'd be able to possibly switch into the PhD program after a year as well, hence why I think this might be the case. Obviously I can't say for sure though.

P-representation
Posts: 61
Joined: Fri Dec 23, 2011 12:56 am

Re: Unexpectedly rejected almost everywhere, Please explain..

Post by P-representation » Sun Mar 17, 2013 4:11 pm

Maybe it's because String Theory is a really really difficult field to get into. I've actually seen a similar situation play out last year with someone I knew. He too had pretty remarkable profile (stellar GPA, reco from a top string theorist in my country) but he couldn't get in anywhere (including Stony Brook and a couple of even lower ranked programs nor the PSI program in Canada). Not that I'm anyone to give advice, but maybe you'd want to apply again in a slightly different research area in Physics? That's what my friend did and he managed to get into a top 10ish school this time.

eclaire1
Posts: 6
Joined: Fri Mar 01, 2013 12:14 am

Re: Unexpectedly rejected almost everywhere, Please explain..

Post by eclaire1 » Sun Mar 17, 2013 4:36 pm

Sorry to hear about this. My best guess is that P-representation is right - your proposed subfield is extremely difficult to get accepted into. In any case, good luck and hope you get accepted to Santa Barbara!

TakeruK
Posts: 941
Joined: Mon Jan 02, 2012 3:05 pm

Re: Unexpectedly rejected almost everywhere, Please explain..

Post by TakeruK » Sun Mar 17, 2013 4:47 pm

I think you have a pretty good profile, but like bfollinprm said, you are applying to top schools in a very competitive field! In addition, you are an international student, which makes it harder to get in, I think, since we cost more than domestic students!

To answer your question, here are some things about your profile that might have contributed to the results. Of course, it's just my opinion -- people can't know for sure, but it's what you asked for :) Also, I am purposely trying to pick out flaws (as you asked) so this post might be pretty negative, but I don't mean to put you down, just pointing out potential flaws.

1. I would first suspect things that aren't immediately obvious by this profile and/or things that are not quantifiable. The first non-quantifiable thing, but probably the most important in applications, is your LORs. I think it's very unusual for you to have submitted 6 LORs. I don't think that LORs "stack" -- instead they "average out". It sounds like some of your letters are definitely not as strong as others (i.e. the letters you submitted from graduate course instructors). I think 3 strong letters are much better than 3 strong letters + 3 mediocre letters (a "did well in class" letter is always mediocre). I guess it's also a possibility that your research letters were not as strong as you think they were. Getting a letter from a well known prof is only helpful if they are able to provide meaningful comments. Sometimes well-known/busy profs with big labs don't get to know their students as well and write more standard letters that aren't as helpful. I don't know the relationship you had with your letter writers, but the LORs might have been part of the result.

2. The other non-quantifiable thing is your SOP / anything else you wrote in your application. Maybe you were not able to express your goals and research interests clearly enough to demonstrate a good fit with the department. I don't know if you were explaining all these details here on this post just for our benefit, but if you wrote about the same thing (or even more detailed) in your SOP, you might have been oversharing, in my opinion, for your application. Or maybe you didn't communicate it well and it sounds like you were trying to make excuses for things that don't really need excuses (or can't be excused). I'm also not sure why you felt you needed to attach a letter from your school's medical office to document your illness in your last year. If you still managed to get As in your last year, the letter is probably not necessary -- I would only think such a letter is necessary if you failed an entire year due to illness or had to take a break from school (but even so, just saying that you had medical issues is probably enough). Of course, perhaps you are not telling us everything here and maybe you had another good reason to include your medical letter, but I'm just going with the info I have here. In addition, you put your grad courses and "interesting undergrad courses" under "special bonus points" here. It's cool that you have grad courses but it usually won't make much of a difference in grad school admissions. I am not sure what's so interesting about the two undergrad courses you listed, either. So if you focused your SOP on the "wrong" things, that might have been a contributing factor. Things like your grad courses will show up on your transcript -- save your SOP space for research related things!

3. Your grades in non-Physics courses might also be a flag! I agree that with your A's in later QM classes, the C in Quantum Physics 1 probably doesn't make a difference. The B in EM probably doesn't matter either. But I would raise an eyebrow at the C in Experimental Physics I (unless you got higher grades in later experimental courses) and because you said that your Math and Humanities classes lower your GPA significantly. I'm not sure what you mean by that exactly, but doing poorly in Math (you said As and Bs but what is the split?) isn't a good thing for a physicist (and I would actually count Math courses as part of a Physicist's "major GPA"). In addition, it sounds like you also did not do so well in the other sciences nor the Humanities. I don't think these things alone are a huge deal. However, when combined with your GRE scores, your profile demonstrates someone who is very proficient in coursework and theoretical physics, but you might not be a very well rounded person overall.

4. Finally, maybe this is the most important part, and it ties all of the above together. I would strongly suspect that the main reason you did not get into these schools despite a strong profile (good grades and research experience) is probably due to a lack of fit. Related to what bfollinrpm said and point #2 above, you are applying to a very narrow research field and maybe the profs you wanted to work with did not have the resources to take you on, or that you did not clearly identify your interests / show a good enough fit for them to want to take you on. Maybe, since the field is so narrow, there were enough other people higher up on the list that got the spots instead. Your profile as presented here also shows that you are a "risky" student to take on. Sure, you might have great grades in Physics, but it's hard to say what your other skills (that are required for success in academia) are, especially with less than great performance outside something so narrow as Physics. I think your strange application (6 LORs, a medical letter etc.) and perhaps combined with a lack of demonstrated fit might have shown that you don't know what you were doing in your application and/or you don't know what you want to do in grad school. Again, this is only a guess -- I don't mean to offend!

Overall, I think your goal might have been too narrow (in terms of the field and the calibre of schools you picked) so despite having a pretty good profile, the "narrowness" might have really raised the competition level! I definitely think you are suitable for graduate studies in Physics, but just not the specific programs you applied to this year. Also, there are still some schools that might be still taking applications / looking for good students.

I think it's a good idea that you asked for feedback from the schools. I know some schools will do this but they generally wait until after the entire application season is over (e.g. May?) so ask again in a few months if you don't hear back from them! If you don't mind, you could post their response here, since it might help others. However, sometimes the responses aren't very helpful, and they might not tell you something like "your LORs were not so great" if they want to protect the LOR writers!

kangaroo
Posts: 130
Joined: Fri Jan 13, 2012 5:31 am

Re: Unexpectedly rejected almost everywhere, Please explain..

Post by kangaroo » Sun Mar 17, 2013 11:55 pm

Dude, 6 letters was a TOTAL overkill. I mean besides the good points that TakerUK made, I think you came off as a tad over-earnest with all your insane, incessant padding. And I wouldn't be surprised if that sentiment came through strongly in your SOP as well. Believe it or not, that can be a turn-off. Grad school applications are like a relationship, desperation STINKS.

jigsaw
Posts: 40
Joined: Sun Mar 17, 2013 10:59 am

Re: Unexpectedly rejected almost everywhere, Please explain..

Post by jigsaw » Mon Mar 18, 2013 12:33 am

Thanks a lot for all your reply. I think I should clarify some points which will make it easier for you to analyze :
bfollinprm wrote:There also isn't really a safety in that list
I thought applying to a lot of good schools is just as good a strategy, my reasoning was something like this "I may not get into any particular school, but if I apply for lets say 10 schools, I'll get into one or the other". But in retrospect I don't think it was a good strategy. As for safety schools, I thought I had some in the list - namely UMich, UT Austin, Stony Brook, Rutgers - but in retrospect I think I was being overconfident and perhaps UMich & UT Austin wasn't a safety school for me after all. But do you honestly think that even Stony Brook wasn't a safety for me?
bfollinprm wrote:HEP-TH is very competitive
Do you mean some fields (like hep-th) are more competitive than others? Wow, somehow I didn't know that! Can you please explain why is it so?
TakeruK wrote:In addition, you are an international student, which makes it harder to get in, I think, since we cost more than domestic students!
I haven't thought about it earlier, but you are right for public universities (since there is the extra non-resident tuition part). But does it matter for private universities?
TakeruK wrote:I would first suspect things that aren't immediately obvious by this profile and/or things that are not quantifiable. The first non-quantifiable thing, but probably the most important in applications, is your LORs. I think it's very unusual for you to have submitted 6 LORs. I don't think that LORs "stack" -- instead they "average out". It sounds like some of your letters are definitely not as strong as others (i.e. the letters you submitted from graduate course instructors). I think 3 strong letters are much better than 3 strong letters + 3 mediocre letters (a "did well in class" letter is always mediocre). I guess it's also a possibility that your research letters were not as strong as you think they were. Getting a letter from a well known prof is only helpful if they are able to provide meaningful comments. Sometimes well-known/busy profs with big labs don't get to know their students as well and write more standard letters that aren't as helpful. I don't know the relationship you had with your letter writers, but the LORs might have been part of the result.
I didn't know '3 strong letters are much better than 3 strong letters + 3 mediocre letters', somehow I thought just the opposite and thats why I submitted as many recommendations as a grad school allows - may it be 3, 4 or 6; but now it looks like you are right. Only the 6th rec (when the grad school allows it) was a research recommendation, that too not in my field of interest (it was in astro), the 5th rec (when the grad school allows it) was from a professor (hep-ex) who knows me for years (initially as a student and later as grader), the top 4 (the ones from hep-th professors) were from graduate & advanced undergraduate classes (since I couldn't do research in hep-th). So you may be right, perhaps my recommendations weren't very stellar.
TakeruK wrote:The other non-quantifiable thing is your SOP / anything else you wrote in your application. Maybe you were not able to express your goals and research interests clearly enough to demonstrate a good fit with the department. I don't know if you were explaining all these details here on this post just for our benefit, but if you wrote about the same thing (or even more detailed) in your SOP, you might have been oversharing, in my opinion, for your application. Or maybe you didn't communicate it well and it sounds like you were trying to make excuses for things that don't really need excuses (or can't be excused). I'm also not sure why you felt you needed to attach a letter from your school's medical office to document your illness in your last year. If you still managed to get As in your last year, the letter is probably not necessary -- I would only think such a letter is necessary if you failed an entire year due to illness or had to take a break from school (but even so, just saying that you had medical issues is probably enough). Of course, perhaps you are not telling us everything here and maybe you had another good reason to include your medical letter, but I'm just going with the info I have here. In addition, you put your grad courses and "interesting undergrad courses" under "special bonus points" here. It's cool that you have grad courses but it usually won't make much of a difference in grad school admissions. I am not sure what's so interesting about the two undergrad courses you listed, either. So if you focused your SOP on the "wrong" things, that might have been a contributing factor. Things like your grad courses will show up on your transcript -- save your SOP space for research related things!
I was quite clear about my goals and research interests in my SOP. Almost all of the explanations & excuses here are for you guys to understand the situation better, I did not 'overshare' in my SOP. As I wrote earlier, due to illness I had to switch from focused option of Physics major to the flexible option, as a consequence of which I didn't have a senior thesis, which I thought was a big deal (infact the biggest weakness in my application in my opinion) and needed explanation -- I attached the letter from MIT Medical just to show that I wasn't making excuses. I thought completing graduate courses (including QFT & GR) demonstrates that I can handle graduate school (despite my low overall GPA); and those two undergraduate classes were among the factors that contributed to my decision of going for hep-th, which makes them 'interesting'. Again, I didn't make a big deal of it in my SOP; as you said, they are already in my transcript.
TakeruK wrote:Your grades in non-Physics courses might also be a flag! I agree that with your A's in later QM classes, the C in Quantum Physics 1 probably doesn't make a difference. The B in EM probably doesn't matter either. But I would raise an eyebrow at the C in Experimental Physics I (unless you got higher grades in later experimental courses) and because you said that your Math and Humanities classes lower your GPA significantly. I'm not sure what you mean by that exactly, but doing poorly in Math (you said As and Bs but what is the split?) isn't a good thing for a physicist (and I would actually count Math courses as part of a Physicist's "major GPA"). In addition, it sounds like you also did not do so well in the other sciences nor the Humanities. I don't think these things alone are a huge deal. However, when combined with your GRE scores, your profile demonstrates someone who is very proficient in coursework and theoretical physics, but you might not be a very well rounded person overall.
I didn't take any other experimental class, but why do you think it matters? Afterall, I'm going for hep-th, not hep-ex. By 'Humanities classes lowered my GPA significantly' I meant that while my grades in Physics courses were mostly A's (5.0/5.0), in other subjects I mostly received B's (4.0/5.0) - and there were quite a few of them, which brought my overall GPA close to 4.5/5.0 - significantly lower than my major GPA. I calculated my major GPA using only Physics courses (8.something), and I got 4.7/5.0, if I include Math courses (18.something) as well, it would be 4.6/5.0 instead - lower, but not too lower. Besides, are you sure B's are that bad at MIT? I don't know about others, but I had to work a lot to earn even those B's, so I wouldn't call it 'doing poorly'. I accept that I'm not a well rounded person, but I thought such things are relevant to college, not grad school. It may be my misconception, but I thought only Major GPA matters in case of grad school admissions, not overall GPA.
TakeruK wrote:Finally, maybe this is the most important part, and it ties all of the above together. I would strongly suspect that the main reason you did not get into these schools despite a strong profile (good grades and research experience) is probably due to a lack of fit. Related to what bfollinrpm said and point #2 above, you are applying to a very narrow research field and maybe the profs you wanted to work with did not have the resources to take you on, or that you did not clearly identify your interests / show a good enough fit for them to want to take you on. Maybe, since the field is so narrow, there were enough other people higher up on the list that got the spots instead. Your profile as presented here also shows that you are a "risky" student to take on. Sure, you might have great grades in Physics, but it's hard to say what your other skills (that are required for success in academia) are, especially with less than great performance outside something so narrow as Physics. I think your strange application (6 LORs, a medical letter etc.) and perhaps combined with a lack of demonstrated fit might have shown that you don't know what you were doing in your application and/or you don't know what you want to do in grad school. Again, this is only a guess -- I don't mean to offend! Overall, I think your goal might have been too narrow (in terms of the field and the calibre of schools you picked) so despite having a pretty good profile, the "narrowness" might have really raised the competition level!
Just to make it clear, I did not apply for String Theory only, that main & side thing in field of interest was a clarification for you guys, I applied for hep-th in general, although I did mention in my SOP that I'd be most interested in those 3 subfields - so it isn't as narrow as you make it sound like. And I'm pretty sure that I was quite clear about 'what I want to do in grad school' and why I'm interested in a particular graduate school (if thats what you mean by 'demonstrated fit') in my SOP. Also, can you please elucidate this -- "Your profile as presented here also shows that you are a "risky" student to take on. Sure, you might have great grades in Physics, but it's hard to say what your other skills (that are required for success in academia) are, especially with less than great performance outside something so narrow as Physics." -- because I didn't quite get the point.
TakeruK wrote:To answer your question, here are some things about your profile that might have contributed to the results. Of course, it's just my opinion -- people can't know for sure, but it's what you asked for :) Also, I am purposely trying to pick out flaws (as you asked) so this post might be pretty negative, but I don't mean to put you down, just pointing out potential flaws.
Don't worry, I didn't take it personally. I know my application isn't flawless, and thats why I don't expect to get accepted at all of these schools, but from a neutral standpoint (i.e. not trying to pick up flaws only :) ) do you really think my application was so weak that I didn't have a chance of getting into even a single top 10 program, not even in 11-25 type programs?
TakeruK wrote:I think it's a good idea that you asked for feedback from the schools. I know some schools will do this but they generally wait until after the entire application season is over (e.g. May?) so ask again in a few months if you don't hear back from them! If you don't mind, you could post their response here, since it might help others. However, sometimes the responses aren't very helpful, and they might not tell you something like "your LORs were not so great" if they want to protect the LOR writers!
"While we are aware of difficulties beyond your control that you have had to endure during your studies, your course grades must be a part of our decision making process. Just to give you an example, you obtained a grade of C in Quantum Physics I - it is quite possible that this was due to ill health, and it is also possible that under ideal circumstances you may have obtained an A, but that is still a 'maybe', and this is a disadvantage compared to a competing application with straight A's. One other specific suggestion I can make is to further narrow down your area of research interest in your statement of purpose." -- UT Austin

"In your case I can say that HET is just incredibly competitive and all has to be pretty much perfect. It was deemed that it was not." -- UMich

"Since we will have an entirely new Admissions Committee for 2014-15, you may fare better on your next try." -- UChicago

"We rated your application as a self-professed theorist rather severely. We have few theory positions with support this year, and we had to therefore raise our cut. I don't think you could have done anything different to improve your chances except publish more, do more research, etc. all of which is somewhat unrealistic for an undergraduate in theory." -- Stony Brook U

"the real issue is not the weakness of your application but the exceptionally high ratio of the number of very good applicants to the number of places (alas, this is much worse for people from outside the UK/EU). For illustration - we were not able to offer a place to candidates who had offers from Princeton, Caltech and other top places." -- Oxford U
Last edited by jigsaw on Wed Mar 20, 2013 3:42 pm, edited 3 times in total.

bfollinprm
Posts: 1203
Joined: Sat Nov 07, 2009 11:44 am

Re: Unexpectedly rejected almost everywhere, Please explain..

Post by bfollinprm » Mon Mar 18, 2013 1:24 am

jigsaw wrote:Thanks a lot for all your reply. I think I should clarify some points which will make it easier for you to analyze :
bfollinprm wrote:There also isn't really a safety in that list
I thought applying to a lot of good schools is just as good a strategy, my reasoning was something like this "I may not get into any particular school, but if I apply for lets say 10 schools, I'll get into one or the other". But in retrospect I don't think it was a good strategy. As for safety schools, I thought I had some in the list - namely UMich, UT Austin, Stony Brook, (and one last school I didn't name here - that is the only school I got accepted at so far, not a top 20 school but a top 30 school) - but in retrospect I think I was being overconfident and perhaps UMich & UT Austin wasn't a safety school for me after all. But do you honestly think that even Stony Brook wasn't a safety for me?
bfollinprm wrote:HEP-TH is very competitive
Do you mean some fields (like hep-th) are more competitive than others? Wow, somehow I didn't know that! Can you please explain why is it so?
1. No. Safety schools should have a large number of professors in your field of interest, solid financing, and low rankings relative to your fit schools. Stony Brook marginally fits the third criterion (not really in my opinion, or I bet the opinion of the school's admission committee), but it's definitely not huge, and evidently has financing issues. A better choice might have been UVA, which is large, well-funded, and has a significant HEP-TH (and related CMT) representation in their faculty, while being ranked outside of the top 25.

2. It comes down to money. Grants to study string theory are hard to come by, and the money that does come funds postdocs and faculty summer research. The school, by accepting you, is signing on to providing TA funding for the 5-6 years that it will take to get your PhD. Also, theory faculty in general have to be much more hands-on with their grad students, and therefore take fewer students at the same time. Mostly though, I think it has to do with money. A top school can effectively support 4-5 experimental condensed matter physicists with the support they have to offer you, since experimental physicists rarely have to stay on TA-ships longer than a year or so.

The above is true for theory in general, but HEP-TH is actually worse. Discussions I've had with HEP-TH and HEP-EX grad students and postdocs say this is because the large experimental collaborations like ATLAS and CMS, with ~1k people in them each, produce an excessive overpopulation of postdocs, both in experiment and theory (since theory-interested grad students work in these collaborations too in order to get better access to the data and funding). As a result, the competition for post-doc positions is very high, which sucks up even more of the limited funds to pay researchers, which makes the share of the pie for grad students even lower. Essentially, there are not enough faculty positions to support the scope of the current HEP community, and that's begun to cause a squeeze at the admissions level in HEP-TH.
TakeruK wrote:In addition, you are an international student, which makes it harder to get in, I think, since we cost more than domestic students!
I haven't thought about it earlier, but you are right for public universities (since there is the extra non-resident tuition part). But does it matter for private universities?
No, not directly. But the number (and quality) of international students applying for private universities is very high, and even private universities want to keep a certain percentage of domestic acceptances, likely (though this is conjecture) in an attempt to keep the character of the department American.

TakeruK wrote:I would first suspect things that aren't immediately obvious by this profile and/or things that are not quantifiable. The first non-quantifiable thing, but probably the most important in applications, is your LORs. I think it's very unusual for you to have submitted 6 LORs. I don't think that LORs "stack" -- instead they "average out". It sounds like some of your letters are definitely not as strong as others (i.e. the letters you submitted from graduate course instructors). I think 3 strong letters are much better than 3 strong letters + 3 mediocre letters (a "did well in class" letter is always mediocre). I guess it's also a possibility that your research letters were not as strong as you think they were. Getting a letter from a well known prof is only helpful if they are able to provide meaningful comments. Sometimes well-known/busy profs with big labs don't get to know their students as well and write more standard letters that aren't as helpful. I don't know the relationship you had with your letter writers, but the LORs might have been part of the result.
I didn't know '3 strong letters are much better than 3 strong letters + 3 mediocre letters', somehow I thought just the opposite and thats why I submitted as many recommendations as a grad school allows - may it be 3, 4 or 6; but now it looks like you are right. Only the 6th rec (when the grad school allows it) was a research recommendation, that too not in my field of interest (it was in astro), the 5th rec (when the grad school allows it) was from a professor (hep-ex) who knows me for years (initially as a student and later as grader), the top 4 (the ones from hep-th professors) were from graduate & advanced undergraduate classes (since I couldn't do research in hep-th). So you may be right, perhaps my recommendations weren't very stellar.
Yes, it's a very bad decision to put course recs ahead of research recs, even if your research was in biophysics filling pipets.

TakeruK wrote:Your grades in non-Physics courses might also be a flag! I agree that with your A's in later QM classes, the C in Quantum Physics 1 probably doesn't make a difference. The B in EM probably doesn't matter either. But I would raise an eyebrow at the C in Experimental Physics I (unless you got higher grades in later experimental courses) and because you said that your Math and Humanities classes lower your GPA significantly. I'm not sure what you mean by that exactly, but doing poorly in Math (you said As and Bs but what is the split?) isn't a good thing for a physicist (and I would actually count Math courses as part of a Physicist's "major GPA"). In addition, it sounds like you also did not do so well in the other sciences nor the Humanities. I don't think these things alone are a huge deal. However, when combined with your GRE scores, your profile demonstrates someone who is very proficient in coursework and theoretical physics, but you might not be a very well rounded person overall.
I didn't take any other experimental class, but why do you think it matters? Afterall, I'm going for hep-th, not hep-ex.
Experimental ability matters in physics, if not to your potential advisor in particular, definitely to the committee as a whole. If you'd have applied to math programs (where string theory faculty are here at Davis, for instance) maybe it wouldn't have mattered. The math classes might have mattered more, though. It's not like you have a huge HEP-TH research or coursework* background from undergrad, so the things that are directly important are not well probed in your application, so the school has to go on proxies, be they fair or not.

TakeruK wrote:Finally, maybe this is the most important part, and it ties all of the above together. I would strongly suspect that the main reason you did not get into these schools despite a strong profile (good grades and research experience) is probably due to a lack of fit. Related to what bfollinrpm said and point #2 above, you are applying to a very narrow research field and maybe the profs you wanted to work with did not have the resources to take you on, or that you did not clearly identify your interests / show a good enough fit for them to want to take you on. Maybe, since the field is so narrow, there were enough other people higher up on the list that got the spots instead. Your profile as presented here also shows that you are a "risky" student to take on. Sure, you might have great grades in Physics, but it's hard to say what your other skills (that are required for success in academia) are, especially with less than great performance outside something so narrow as Physics. I think your strange application (6 LORs, a medical letter etc.) and perhaps combined with a lack of demonstrated fit might have shown that you don't know what you were doing in your application and/or you don't know what you want to do in grad school. Again, this is only a guess -- I don't mean to offend! Overall, I think your goal might have been too narrow (in terms of the field and the calibre of schools you picked) so despite having a pretty good profile, the "narrowness" might have really raised the competition level!
Just to make it clear, I did not apply for String Theory only, that main & side thing in field of interest was a clarification for you guys, I applied for hep-th in general, although I did mention in my SOP that I'd be most interested in those 3 subfields - so it isn't as narrow as you make it sound like. And I'm pretty sure that I was quite clear about 'what I want to do in grad school' and why I'm interested in a particular graduate school (if thats what you mean by 'demonstrated fit') in my SOP. Also, can you please elucidate this -- "Your profile as presented here also shows that you are a "risky" student to take on. Sure, you might have great grades in Physics, but it's hard to say what your other skills (that are required for success in academia) are, especially with less than great performance outside something so narrow as Physics." -- because I didn't quite get the point.
I think he means lack of HEP-TH research, which other students applying to the above schools will have. The important thing to remember is that you're qualified, and most places you applied to will agree, but there is an over-preponderance of qualified applicants, so some qualified students have to be rejected. TakeruK is correctly pointing out that given the choice, admissions at top 15 schools will go for the safe options (which will mean students with demonstrated research ability and--to a lesser extent--wide and general success in undergrad) over the riskier choices.
TakeruK wrote:To answer your question, here are some things about your profile that might have contributed to the results. Of course, it's just my opinion -- people can't know for sure, but it's what you asked for :) Also, I am purposely trying to pick out flaws (as you asked) so this post might be pretty negative, but I don't mean to put you down, just pointing out potential flaws.
Don't worry, I didn't take it personally. I know my application isn't flawless, and thats why I don't expect to get accepted at all of these schools, but from a neutral standpoint (i.e. not trying to pick up flaws only :) ) do you really think my application was so weak that I didn't have a chance of getting into even a single top 10 program, not even in 11-25 type programs?
You had a chance, but it wasn't a guarantee. I'd say I'd be pretty surprised if you were a domestic applicant, mildly surprised given you did your undergrad at a respected US institution, and would understand surprise in someone else if you'd have been an international student who studied as an undergrad in Europe or Asia.



*(to be clear, undergrad level HEP 'theory' just represents the required background experimentalists need to do their experiments, not the theoretical background you need to make progress in advancing the theory)

Robespierre
Posts: 30
Joined: Mon Jan 14, 2013 4:22 pm

Re: Unexpectedly rejected almost everywhere, Please explain..

Post by Robespierre » Mon Mar 18, 2013 2:05 am

If hep-th is so competitive, like medical school or something (I know it's the most competitive in physics, but I'm not sure of the exact extent. Does anyone have concrete statistics?), then a 4.4 overall GPA (corresponding to a 3.4 at other schools, if I remember correctly) will probably not be up against the competition, even for an applicant from MIT. Then again, there does not seem to be any literal 'competition' among physics/hep-th students.

TakeruK
Posts: 941
Joined: Mon Jan 02, 2012 3:05 pm

Re: Unexpectedly rejected almost everywhere, Please explain..

Post by TakeruK » Mon Mar 18, 2013 2:28 am

Glad I was helpful -- thanks for clarifying!

After seeing your post, I am mostly sure that the main reason is the lack of research related LORs! I'm sorry that you didn't have a mentor/advisor to help you with your application -- surely they would have caught onto the fact that you were submitting mostly course related LORs when you had some research experience background. I know at some of the top schools, you would be limited to only 3 letters so if they were all course related ones....then it's a major disadvantage! Many would agree that a "did well in courses" letter should only be used if you don't have 3 research LORs. Like bfollingrpm said, the research LORs should show that you have research experience (in any capacity). Being simply exposed to research is important -- the actual work done (or field it's in) is not! I

To answer your other questions you posed:

1. The advice and prediction I got from profs in Canada familiar with the US system was that I would get into at least one top private university but be rejected from most of the lower ranked public universities because of funding problems for international students. And they were right! You can see this from their stats. For example, the UC schools only have a 10% international student population while the incoming grad class at Caltech in 2012 was 40% international. Usually if the tuition rate is a set amount, no matter if you're domestic or international, it's a good sign that being international won't hurt you, in terms of increased funding costs. The other ways being international could hurt you is that your undergrad may not be as well known, but since you're from MIT, that shouldn't be a problem!!

2. I wouldn't think Bs at MIT are bad. In your original post, it sounded like you got much lower grades but what you said sounds okay. From the feedback from some of your schools, it does look like the Cs are harmful though. You might have gotten As in the grad level QM classes, but from my experience, it's much easier to get an A in grad school than undergrad (even if this is not true at MIT, this might be true at the schools you're applying to) so As in those classes might not count as much. This is because the primary focus of a grad student is to do research, not classes. So, grad classes have far lower expectations -- they are there just to make sure the student has a decent foundation. Profs don't want their grad students spending > 10 hours a week studying for their course when they would rather their student spend that time in the lab/doing research. On the other hand, courses are basically the meat of the undergrad workload. This advice may be too late for you, but to anyone else, I would strongly advise against taking too many (5 is probably too many) graduate courses while you're an undergrad. Take them if they are directly related to your research, but you would probably do better if you invest that time in doing better in undergrad courses, or doing a research project.

3. As for my comment on the "risky" nature of your application, what I meant is that you sound like someone who looks good "on paper" but you don't have much experience to back it up. The strongest parts of your application are your high scores in Physics courses and tests and letters to back that up. However, as I said above, grad schools don't necessarily want someone who is good at taking courses. They want someone who can produce research results and come up with ideas for new research. If all they see is someone who got good grades but no letters to show research then it's risky. This is related to why I brought up the Experimental Physics grade and the Humanities grades. These topics are not related to your specified field of interest but the lower grades there (especially in Experimental Physics) shows that you might have a pretty limited skillset. It shows that you really do excel that the things you are familiar with but that you might struggle a lot when outside of your comfort zone. I think when you are doing research, you will be outside of your comfort zone more often than in a comfort zone! Usually, I would not say GRE scores are very important, but in this case, the big difference in your quantitative/physics scores vs. "communication" scores also add to this perception. I'm not saying you need to redo your GREs though -- the current scores might amplify a potential risk expressed by the other parts of your profile, but if those are addressed, then the GRE scores will probably just be ignored.

One of the schools gave you the feedback that your stated research interest was not specific enough, but you also said that you thought your SOP clearly said what you wanted to do in grad school and why you wanted to do it. So, I think this might be another issue! Maybe your SOP is not as clear as you thought it was. Clear communication is very important in science!

So, to your last question, yes, I think with the unfortunate choice of LORs and perhaps a misunderstanding of your SOP, that is probably enough to weaken your application to put you out of the running for the top schools. Your major GPA is strong though! Everyone has a different opinion, but I think LORs (and the research experience they describe) carry the most weight, followed by how well your SOP describes your fit, then grades/course experience, then test scores, then other aspects of your writing in your SOP, maybe.

I'm glad you did get into one school though, and a Top 30 school that fits you well is probably still better than a Top 10 school that doesn't suit you! I guess you will have to decide if you want to attend that school or try again next year for another school. I don't really know what is the best path for you. But, if you do want to try again, it sounds like you really need to revise your LOR selection and I think you should rewrite your SOP too. Also, you should try to find a prof who can mentor you through the application process. You didn't know about the LOR thing, and there may be other "unwritten rules" about academia that you aren't aware of either! I was the same way when I first started -- I am the first person in my family to go to University so I really relied on my advisors to be my "academic parent" and guide me through the world of academia. Without my mentors, I would have totally screwed up my first round of applications! If you don't have contact with your MIT profs anymore, there has been several books on this topic published in the past few years. I haven't read them so I don't have a recommendation though!

And, if you do want to reapply next year, maybe someone at MIT will hire you as a research assistant for the year so that you can have at least 2 research related LORs. For the top schools in your list, I would predict that almost all the applicants have submitted at least 2 research related LORs! Again, maybe you don't want to / need to reapply, but if you do, you can take heart in the fact that the two things that need the most attention (rearrange LORs and maybe rewrite SOP) are things completely in your control. For issues like GPA, it would generally not be possible to go back in time and change your grades :P
Last edited by TakeruK on Mon Mar 18, 2013 2:33 am, edited 1 time in total.

Kinbote
Posts: 24
Joined: Tue Feb 12, 2013 7:30 pm

Re: Unexpectedly rejected almost everywhere, Please explain..

Post by Kinbote » Mon Mar 18, 2013 2:32 am

I initially believed I was in a similar situation - seemingly good enough everything to get into a top 10 school, and yet rejected.
I contacted faculty member's from most of these institutions and got these responses --

MIT : "Your application is as strong as those of the student's I accepted. However, their research interests better align with mine and it would be to your benefit to consider another institution where the faculty member's are involved in the research that you wish to pursue."

Stanford : "The faculty member you indicated strongest interest in working with currently has 5-6 graduate students and is not looking to take any more."

Harvard : I contacted a faculty member and he said he'd put me on the waitlist and that I may attend if I can secure my own funding.

I quote these here to show that "being good enough" is not sufficient criteria to be accepted to any top 20 school. There are many other considerations that go into these decisions -
1. International students cost more at state universities. As you say, this does not apply to private universities.

2. Due to the volume of qualified applicants, school's can't accept just qualified students. Rather, they accept qualified students whose research interest aligns nicely with that of a faculty member.

3. You should know your research interest really well - not just broadly. So if there is an applicant with your identical credentials who says "interested in topological insulators" versus just "condensed matter theory," that person gets preference. The argument against this is that undergrads shouldn't be expected to know specifically what they want to do but if certain applicants do, then they have a distinct advantage.

4. Then there is the question of whether the faculty member you listed is currently taking students or not. Again, you'll choose faculty members based on research interests and you have to be careful while making this selection. Also, these faculty members should have the funds to take you on as a research assistant.

Maybe 8-10 years ago, having a publication would have almost guaranteed a spot in a top 10 school but this is no longer the case, just as having a 990 GRE score doesn't really add to your application if you're an international student.

Also, as kangaroo mentioned, 6 recs may be overkill. Faculty don't spend hours looking at each application, so one very good recommendation that says something more substantial than "this is a good student" will catch their eye and interest them, rather than 4-5 recommendations that say just that.

Let me also add that while I was initially dissatisfied with my acceptance at Boulder, I have since received acceptances from UPenn and UCSB, and despite that, I'm now definitely leaning towards Boulder given that they have some extremely good people (as bfollinprm was kind enough to point out) - so you shouldn't be disheartened with your acceptance. If it's a top 30 school and you can find faculty who do research you're interested in, you can definitely get a good post-doc.

I know someone in math who is absolutely fantastic but hasn't gotten in anywhere, so your rejections don't indicate that you're a poor applicant - only that what you indicated as research interests didn't line up perfectly with what these schools were looking for.

esopterodactyl
Posts: 22
Joined: Tue Jan 31, 2012 3:54 pm

Re: Unexpectedly rejected almost everywhere, Please explain..

Post by esopterodactyl » Mon Mar 18, 2013 4:20 am

I agree with much of what has already been posted. Frankly, you are the sort of candidate who I would expect to be getting in to these top 20 schools.
In case you haven't been looking at the admissions profiles recently, I was rejected from the 4 top 20 schools I applied to.
I love physics and honestly I would be extremely happy at this point just to get in anywhere. Sure I'm disappointed about the rejections, but I know it's a statement about admissions standards and competition more than my ability.
I wish you the best of luck and hope you have success at whichever institution you end up.

nerv
Posts: 7
Joined: Fri Feb 22, 2013 5:36 pm

Re: Unexpectedly rejected almost everywhere, Please explain..

Post by nerv » Mon Mar 18, 2013 4:37 am

As much as grad school is an investment to the student, the student is also a huge investment to the school. This includes money, of course, but also time (5+ years) and effort spent by the faculty on the student. So, you need to understand what the school is looking for in a potential PhD student. What they're looking for is someone who can perform research, independently, and who can produce meaningful results in the field of interest. What they're looking for is not someone who is good at taking classes (remember, a good student might not be a good researcher). Thus, the most important thing to present of yourself is your ability to conduct research on your own, whether this is shown in your SOP or LORs. Using this piece of information, the school then tries to match you up with faculty who might be interested in working with you (i.e. fit).

From what I've read of your situation, it seems you put your focus on the wrong thing. Your LORs are all course-based, and also you spent a big chunk of your post explaining your grades, which is not looked too carefully upon by the committee unless there's a red-flag like a C or D. (The C in experimental physics on your part is actually a red flag - just because you are a theorist doesn't mean you need not understand experiments). As many people here have said, research-based LORs >>> course-based LORs, and this is one area where it definitely hurt you. I don't know about your SOP, but any evidence backing your desire to go into HEP-th would be helpful, but there doesn't seem to be any from your research experience. You also say that you didn't have enough knowledge to do theoretical research as an undergraduate, but I strongly disagree with that - even as an undergraduate, one can pick a direction and work really hard on it, and usually get somewhere after a year of pain and sweat. When I was a sophomore (i.e. just after freshman summer), despite having taken only introductory QM and classical EM both on the level of Griffiths, I (foolhardily) decided to talk to a string theorist in my school and asked if I could work on something related to string theory with him. I didn't know anything then (and not much more now actually, but anyway..), but I ended up trying to read Zwiebach's book on my own, some of Weinberg's and Witten's papers etc on matrix models in string theory. I was really lost all the time, and in the end my prof had to sit down and explain everything to me, and I basically wrote a paper about what he taught me (which was nothing new), but I did learn some QFT, string theory, supersymmetric stuff... The important thing though was that I tried and it showed - I had something tangible which I produced - and I wrote it in my SOP, and I think it helped. (The other important thing was that I found that I didn't like hep at all :P )

Personally, comparing myself to you, I think my grades / classes are the same as you. I have a 3.75 overall GPA, which was caused by a bunch of B+s in humanities courses, and my major GPA was only 3.87 (not a stellar 4.0 like some people have in this forum!!), because of a B in QFT. My PGRE was also 990, Q was 166 and V 167, W 5.0, so in terms of scores and grades I'd say we're pretty evenly matched up. But unlike you I was really worried about my application, because I didn't think my application fared too well in the research aspect, which I knew is the primary aspect. My indicated field of interest is CMT, but I only switched into it fairly late into the game, so I had only done one small (mandatory) piece of independent work on it in quantum phase transitions. How could I compare to someone else who had devoted 2, 3 years of their lives on some project? (There's such a guy in my school and not surprisingly he got into MIT, UCSB, Berkeley etc.) My 3 recommendations were from people I've done research with - one from a researcher at a national lab, one from the professor I did the piece of independent work with, and the last from a professor whom I'd just started my thesis work with. It wasn't the most optimal of recommendations, because the stuff I did in the national lab was totally unrelated to CMT (but I chose him because he liked me and I felt he could write quite a bit about my research skills in general), the stuff I did with the first professor was kind of trivial, and I had only just started working with the second professor who hardly knew me at all! I thought I had a shot somewhere but that it'd most likely be only 1 or 2 schools (my list is once again, largely similar to yours). Turned out that my hunch was right, got rejected by all except 2, but luckily one of the two was UIUC which is absolutely fabulous for CMT so I'm happy with my situation.

I can't stress how important research experience is: I have a friend one year above me, who had a (somewhat abysmal) 3.1 GPA overall. But he still got into Harvard physics, because he did quite significant research with this professor over summer and for his thesis, and I think it's quite certain that the LOR from that professor got him to where he is now. He only spent 2 sentences in his SOP explaining his bad grades - something along the line that he didn't care too much about grades because he feels there's so much more to learning than taking tests, so he spent his time doing other more worthwhile pursuits - not sure if the schools bought that, but evidently the bad grades didn't really matter that much.

quantumcat2002
Posts: 2
Joined: Thu Mar 06, 2008 12:01 pm

Re: Unexpectedly rejected almost everywhere, Please explain..

Post by quantumcat2002 » Mon Mar 18, 2013 11:35 am

In addition to the comments that others have made about your letters of rec and your humanities grades, your writing and verbal GREs are quite low. If you don't get in at your last school I would spend the year seriously studying trying to get those numbers up since that's the thing you can still change and if you managed to post some good numbers there it would help make up for mediocre humanities grades. Physics professors really do need to be able to write. Good luck with your last class and with your apps next year and way to go on the physics GRE.

grndfthrprdx
Posts: 2
Joined: Mon Mar 08, 2010 3:03 pm

Re: Unexpectedly rejected almost everywhere, Please explain..

Post by grndfthrprdx » Mon Mar 18, 2013 1:31 pm

To add to what a few other people have said, it probably looks weird on an application if you have a research experience then don't submit LORs from any research advisors. This might look as if it was a bad experience. Grad school is only about doing research, so having someone say you are competent at research is much more useful than someone (even a famous professor) saying you are competent at coursework.

The reason HEP-TH is so competitive is that there is high supply and low demand. Many top students want to do HEP-TH, and there isn't much funding or many professors.

vttd
Posts: 53
Joined: Wed Jul 15, 2009 1:23 pm

Re: Unexpectedly rejected almost everywhere, Please explain..

Post by vttd » Mon Mar 18, 2013 1:44 pm

I agree with what previous posters have said. The most important aspect of your application is demonstrating that you will be a good researcher. Your SOP should reflect that you will be coming into graduate school knowing what subfield you want to work on and with who. It also helps to scout out if those professors are taking on new students and the easiest way to do that is email them. Most of the professors I emailed while I was applying responded back to me. Your LOR may have also sent an implicit message to the deciding committee that something went astray in your research so you avoided asking those professors to write you letters. It doesn't matter as much if the research is outside your field, most research skills are transferable to any field.

I think from what you have told us your application looks stellar, but nobody is a shoe-in for the top 10. There are many factors that go into the admission process. And while it would be fantastic to get into a top 10 school, not getting into one is not the end of the world. There are plenty of institutions that are not generally ranked high, but are high in the rankings for specific subfields. One of the most important factors for getting a job is who you know. If you go to a top 50 school but your advisor is number 1 in his/her field then the school name doesn't matter as much, that professor will have plenty of connections that will get you in far more places than a university's name.

Take into consideration how good of a fit the school you got into is for you. Generally you shouldn't apply to any school you won't attend (waste of money and time), but I know it is common to want to go SOMEWHERE. Just remember this is your life and you have control over the direction you want to take it in. You may find you actually love the school you got accepted to after the visit

friedrice
Posts: 8
Joined: Fri Dec 28, 2012 3:55 pm

Re: Unexpectedly rejected almost everywhere, Please explain..

Post by friedrice » Mon Mar 18, 2013 3:52 pm

quantumcat2002 wrote:In addition to the comments that others have made about your letters of rec and your humanities grades, your writing and verbal GREs are quite low. If you don't get in at your last school I would spend the year seriously studying trying to get those numbers up since that's the thing you can still change and if you managed to post some good numbers there it would help make up for mediocre humanities grades. Physics professors really do need to be able to write. Good luck with your last class and with your apps next year and way to go on the physics GRE.
I agree with quantumcat2002 that you need to pull up your verbal and writing scores. I did my general GRE twice because my first verbal score was rather low—especially when compared with other American students. I am an international student as well, and since English is not my first language, I felt that I had to prove that I am as good as the other American students. I'd say try to get at least 80% for verbal and writing scores. It's really not that difficult, you just need to practice. You can even prepare for the writing portion. I had some structure in mind for my essay—even before I took the test. That way, I could concentrate more on the content and reasoning.

User avatar
Tom Joad
Posts: 120
Joined: Fri Aug 24, 2012 9:35 pm

Re: Unexpectedly rejected almost everywhere, Please explain..

Post by Tom Joad » Mon Mar 18, 2013 5:14 pm

Could you post your statement of purpose? That would also give us a better illustration of your overall application and lead to more specific advice. Other than everything that's already been said, of course.

User avatar
Skullgrid
Posts: 59
Joined: Sat Aug 21, 2010 6:50 pm

Re: Unexpectedly rejected almost everywhere, Please explain..

Post by Skullgrid » Wed Mar 20, 2013 2:20 am

Apparently he deleted his profile?? I wish he wouldn't have... this thread could be extremely useful to future generations. In my opinion this one might even deserve a sticky.

User avatar
Andromeda
Posts: 127
Joined: Wed Dec 10, 2008 3:17 pm

Re: Unexpectedly rejected almost everywhere, Please explain..

Post by Andromeda » Wed Mar 20, 2013 11:12 am

Wow, paranoid much huh...

It honestly kinda irks me when people do stuff like that. You want everyone's help and get feedback from others who have posted their profiles, but that only works if people are willing to do so in turn.

*gets off of soapbox*

TakeruK
Posts: 941
Joined: Mon Jan 02, 2012 3:05 pm

Re: Unexpectedly rejected almost everywhere, Please explain..

Post by TakeruK » Wed Mar 20, 2013 11:52 am

Andromeda wrote:Wow, paranoid much huh...

It honestly kinda irks me when people do stuff like that. You want everyone's help and get feedback from others who have posted their profiles, but that only works if people are willing to do so in turn.

*gets off of soapbox*
I agree! At other forums, e.g. thegradcafe, you can only edit your post within ~30 mins of writing it and once a thread has replies, the moderators won't delete the content. Maybe Physicsgre.com should consider a similar policy!

esopterodactyl
Posts: 22
Joined: Tue Jan 31, 2012 3:54 pm

Re: Unexpectedly rejected almost everywhere, Please explain..

Post by esopterodactyl » Wed Mar 20, 2013 12:13 pm

Do people seriously not know about crawlers anymore?
Just because you delete a post doesn't mean it's erased from the internet. If someone wants that information, they can get it.

I hope this doesn't breed a more intense culture of paranoia on this site, because it's already ridiculous.

No one cares what your profile is like and the people who are able to match your profile to your identity won't do so because of professional concerns and/or standards (until after decisions are made I would assume). Post your info and move on...

jigsaw
Posts: 40
Joined: Sun Mar 17, 2013 10:59 am

Re: Unexpectedly rejected almost everywhere, Please explain..

Post by jigsaw » Wed Mar 20, 2013 2:56 pm

Skullgrid wrote:Apparently he deleted his profile?? I wish he wouldn't have... this thread could be extremely useful to future generations.
Andromeda wrote:It honestly kinda irks me when people do stuff like that. You want everyone's help and get feedback from others who have posted their profiles, but that only works if people are willing to do so in turn.
TakeruK wrote:I agree! At other forums, e.g. thegradcafe, you can only edit your post within ~30 mins of writing it and once a thread has replies, the moderators won't delete the content. Maybe Physicsgre.com should consider a similar policy!
Guys, I sincerely apologize, I never thought someone might be interested in my profile. I came here to get some opinions, and I got some thanks to you guys, and I'm really grateful for that. I thought I was done, so I decided to quit, I deleted my posts, and requested my account to be deactivated. I just saw your posts, and realized my mistake, so I requested my account to be reactivated, and I'm back. I just restored all of my posts (they aren't same word by word, but pretty much the same), I also added more information in my profile (so you may want to take a look once again even if you read it in the past). Some of you were kind enough to send me your opinion by private message as well, if you guys still have them in your sent messages folder, I request you to post them here, so that future members can benefit from your opinion as well (I request you not to post my reply though). I would have done the same, but I deleted the messages before asking my account to be deactivated. To other members, feel free to add your opinion if you haven't done so already.
esopterodactyl wrote:No one cares what your profile is like and the people who are able to match your profile to your identity won't do so because of professional concerns and/or standards (until after decisions are made I would assume). Post your info and move on...
Thanks a lot for your assurances. I was uncomfortable posting some information before, I just added them in my profile.
Skullgrid wrote:In my opinion this one might even deserve a sticky.
You really think so? Thanks my friend, I'm honored.
Last edited by jigsaw on Thu Mar 21, 2013 12:22 am, edited 1 time in total.

NoGodTryScience
Posts: 5
Joined: Tue Sep 04, 2012 2:44 pm

Re: Unexpectedly rejected almost everywhere, Please explain..

Post by NoGodTryScience » Wed Mar 20, 2013 4:51 pm

.
Last edited by NoGodTryScience on Wed Jul 22, 2015 4:39 pm, edited 1 time in total.

blighter
Posts: 256
Joined: Thu Jan 26, 2012 6:30 pm

Re: Unexpectedly rejected almost everywhere, Please explain..

Post by blighter » Wed Mar 20, 2013 5:31 pm

NoGodTryScience wrote:
TakeruK wrote:
Andromeda wrote:Wow, paranoid much huh...

It honestly kinda irks me when people do stuff like that. You want everyone's help and get feedback from others who have posted their profiles, but that only works if people are willing to do so in turn.

*gets off of soapbox*
I agree! At other forums, e.g. thegradcafe, you can only edit your post within ~30 mins of writing it and once a thread has replies, the moderators won't delete the content. Maybe Physicsgre.com should consider a similar policy!
That is kind of silly. How will people update their admission results then? There are profiles on gradcafe but no info on whether people got admitted- precisely because of this policy.
How about if they allow you to only append to posts and not remove anything?

actrask
Posts: 19
Joined: Tue Feb 12, 2013 7:52 pm

Re: Unexpectedly rejected almost everywhere, Please explain..

Post by actrask » Wed Mar 20, 2013 7:11 pm

Wait so you have got into Rutgers?

I don't think you mentioned this in your OP. Rutgers, although maybe not topping the rankings, is still a very good school for hep-th. It's underrated in my opinion.

Although you may think less of it since it was last on your list of "safety" schools, I don't think you should have that many reasons to count it out. You don't really have safety schools for hep-th anyway and you applied to almost all top-10 programs.

TakeruK
Posts: 941
Joined: Mon Jan 02, 2012 3:05 pm

Re: Unexpectedly rejected almost everywhere, Please explain..

Post by TakeruK » Wed Mar 20, 2013 7:29 pm

NoGodTryScience wrote: That is kind of silly. How will people update their admission results then? There are profiles on gradcafe but no info on whether people got admitted- precisely because of this policy.
That is a good point -- and perhaps why gradcafe doesn't seem to have something that is definitively the most useful part of physicsgre.com! Perhaps there is a way to have different edit policies for different message boards (i.e. allow editing for the profile threads but not elsewhere?). If not then I would agree that it's better to allow editing (and the occasional person who would remove their information) than to not have updated profiles! It would probably be better to educate people about the reasons to post the information instead of "enforcing" it in this way.

Godot
Posts: 12
Joined: Wed Feb 13, 2013 1:09 pm

Re: Unexpectedly rejected almost everywhere, Please explain..

Post by Godot » Wed Mar 20, 2013 7:37 pm

i disagree; though it may suck when someone deletes a useful profile, people should have the right to delete personally identifying information even if it is stored by a web crawler somewhere else. Removing that ability would also likely prevent some people from posting that information in the first place.

esopterodactyl
Posts: 22
Joined: Tue Jan 31, 2012 3:54 pm

Re: Unexpectedly rejected almost everywhere, Please explain..

Post by esopterodactyl » Wed Mar 20, 2013 7:54 pm

Just wanted to clear my message up a little - I'm not implying that I want all info to be permanent, uneditable/undeletable, etc.
I was simply pointing out that there is no reason to freak out and delete everything.

jigsaw
Posts: 40
Joined: Sun Mar 17, 2013 10:59 am

Re: Unexpectedly rejected almost everywhere, Please explain..

Post by jigsaw » Thu Mar 21, 2013 12:35 am

actrask wrote:Wait so you have got into Rutgers?

I don't think you mentioned this in your OP. Rutgers, although maybe not topping the rankings, is still a very good school for hep-th. It's underrated in my opinion.

Although you may think less of it since it was last on your list of "safety" schools, I don't think you should have that many reasons to count it out. You don't really have safety schools for hep-th anyway and you applied to almost all top-10 programs.
Yes I added some new information recently, so you may want to take a look at my posts once again. You may remember I wrote something like 'I have a place to go this time, it is a top 30 school, although not top 20' earlier, it was Rutgers I was talking about. I know it is a very decent school for hep-th (or else why would I apply for it), it is just people frown at you when you go to a school not in top 10 (or atleast top 20), and there is inferiority complex associated with going to a school that isn't ranked very high - perhaps you Americans don't feel that way, but this is the mindset of almost all Asians (including me).

TakeruK
Posts: 941
Joined: Mon Jan 02, 2012 3:05 pm

Re: Unexpectedly rejected almost everywhere, Please explain..

Post by TakeruK » Thu Mar 21, 2013 1:36 am

jigsaw wrote: I know it is a very decent school for hep-th (or else why would I apply for it), it is just people frown at you when you go to a school not in top 10 (or atleast top 20), and there is inferiority complex associated with going to a school that isn't ranked very high - perhaps you Americans don't feel that way, but this is the mindset of almost all Asians (including me).
I think this is an attitude that should probably change for you to fit in well in academia.

jigsaw
Posts: 40
Joined: Sun Mar 17, 2013 10:59 am

Re: Unexpectedly rejected almost everywhere, Please explain..

Post by jigsaw » Thu Mar 21, 2013 2:20 am

OK, so let me summarize what has commonly be pointed out by other PhysicsGRE members so far :

(A) HEP-TH (especially String Theory) is extremely competitive already, the fact that I'm an International student (aka 'expensive' for public schools) makes it even more severe. I was aiming a bit too high (I applied for 9 top 10 schools, 2 top 20 schools, and 2 top 30 schools of US; as well as 2 of the top 3 schools in UK), and the schools I considered safety (those in 11-30 range) weren't safety after all (even for a MIT student).

(B) I sent too many recommendation letters (as many as 6 in most cases), which wasn't a good idea. I also put more emphasis on course recommendations (since they were relevant to hep-th and were from well known hep-th professors) than research recommendations (since they were mostly from hep-ex and astro professors), which was a terrible mistake. The recommendations may or may not have been as strong as I thought them to be.

(C) Some also conjectured lack of research experience in hep-th, possibly less clearly demonstrated fit in SOP, low course grade in subjects other than Physics, and low score in GRE Verbal & Writing makes me a 'risky candidates'; and top grad schools may prefer to accept 'safer candidates' instead.

(D) Pure bad luck : 'Statistical Fluctuation', persons of interests no longer taking students, research interest mismatch (as Kinbote pointed out), very limited funding in selected field of interest this particular year, and so on.

Please feel free to suggest if you have any other point in mind.
Last edited by jigsaw on Thu Mar 21, 2013 4:44 am, edited 1 time in total.

jigsaw
Posts: 40
Joined: Sun Mar 17, 2013 10:59 am

Re: Unexpectedly rejected almost everywhere, Please explain..

Post by jigsaw » Thu Mar 21, 2013 2:22 am

Now, I would like to add a few points of my own since I would like to know your detailed opinion about them :

(1) Flexible Option of Physics Major : This option is generally meant for students who aren't sure whether they want to do a graduate study in Physics or in some other field. Although my reasons of doing it was different (already explained), could the admission committee have frowned at the fact that I didn't do the 'Focused Option'? The only major differences are Senior Thesis, and Experimental Physics II. For whatever the reason may be and however well documented it may be, the end result is that I didn't have a Senior Thesis. How severe, if any, an effect can this have on graduate school admission? Unfortunately there is nothing I can do about it now, but still it would be good to know.

(2) Taking a Year Off : As stated earlier, I couldn't afford to study Part III of Math Tripos at University of Cambridge or MSc in Quantum Fields and Fundamental Forces at Imperial College London despite being offered admission (due to lack of financial aid), and had to take a year off. Although I did quite a bit of self study, as you know it is not something that can be 'documented well'. How severe, if any, an effect can this have on graduate school admission? Unfortunately there is nothing I can do about it now, but still it would be good to know.

(3) Health Issues : I missed lots of classes at MIT at various points due to illness. Although I didn't mention it in my SOP (except for the senior year), I don't know if my recommenders did, in any case I had made a full recovery by the time I applied for grad school this year, and I mentioned that clearly in my SOP. Do you think it might have a negative influence on my graduate application if the schools somehow found out about it? If yes, how severe can the effect be?

(4) Limited Math Coursework : I didn't take that many math classes at MIT, especially after my freshman year. I took 2 terms of Calculus, 1 term of Differential Equations, 1 term of Linear Algebra - the theoretical sequence (18.014, 18.024, 18.034, 18.700), not the standard sequence (18.01, 18.02, 18.03, 18.06). Beyond that I learned Math mostly in the context of Physics courses (as much as I needed, and whenever I needed) instead of taking separate Math courses. How severe, if any, an effect can this have on graduate school admission (especially in hep-th which is a little more mathematical than other fields)? Do you think taking GRE Math Subject test would have helped in graduate admission in hep-th? I took GR9768 all of a sudden yesterday, and although I made quite a few silly mistakes, I did manage 88 percentile; with a little practice (taking a few mock tests) I can easily make it 95 percentile (although a 99% will require a little more than that). If I have 95% in Math GRE as well, will it improve my chances if I indeed decide to re-apply (I've yet to decide whether I'll reapply or not)?

(5) Person of Interest in SOP : In my SOP for every school, I quite specifically mentioned which two professors of their school I'm most interested in working with. But I was worried that I may be rejected if those particular professors had no vacancy this year, so to be on the safe side, I wrote down the names of other professors I may be interested in as well in my SOP - this list was somewhat long, 5-10 professors depending on the school. I initially thought this was a good idea, but I'm not so sure anymore after the feedback from UT Austin. What effect, if any, (and how severe) can this have on graduate admission in your opinion?

Please let me know your opinion on the above points, and feel free to add any new point you may have in mind. Thanks.

jigsaw
Posts: 40
Joined: Sun Mar 17, 2013 10:59 am

Re: Unexpectedly rejected almost everywhere, Please explain..

Post by jigsaw » Thu Mar 21, 2013 2:26 am

TakeruK wrote:
jigsaw wrote: I know it is a very decent school for hep-th (or else why would I apply for it), it is just people frown at you when you go to a school not in top 10 (or atleast top 20), and there is inferiority complex associated with going to a school that isn't ranked very high - perhaps you Americans don't feel that way, but this is the mindset of almost all Asians (including me).
I think this is an attitude that should probably change for you to fit in well in academia.
That is a very interesting opinion. Will you kindly elaborate?

blighter
Posts: 256
Joined: Thu Jan 26, 2012 6:30 pm

Re: Unexpectedly rejected almost everywhere, Please explain..

Post by blighter » Thu Mar 21, 2013 3:42 am

jigsaw wrote:
TakeruK wrote:
jigsaw wrote: I know it is a very decent school for hep-th (or else why would I apply for it), it is just people frown at you when you go to a school not in top 10 (or atleast top 20), and there is inferiority complex associated with going to a school that isn't ranked very high - perhaps you Americans don't feel that way, but this is the mindset of almost all Asians (including me).
I think this is an attitude that should probably change for you to fit in well in academia.
That is a very interesting opinion. Will you kindly elaborate?
I don't think there is anything to be elaborated on here. If you feel inferior to the people from top 10, you probably also feel superior to the people outside of to 30. Also I believe you feel inferior because of your ego, as if you are entitled to a top 10 school. People have done well everywhere. I don't think you need to focus on the rankings as long as you get a decent adviser in your field of interest.

jigsaw
Posts: 40
Joined: Sun Mar 17, 2013 10:59 am

Re: Unexpectedly rejected almost everywhere, Please explain..

Post by jigsaw » Thu Mar 21, 2013 4:35 am

blighter wrote:I don't think there is anything to be elaborated on here. If you feel inferior to the people from top 10, you probably also feel superior to the people outside of top 30. Also I believe you feel inferior because of your ego, as if you are entitled to a top 10 school. People have done well everywhere. I don't think you need to focus on the rankings as long as you get a decent adviser in your field of interest.
You are clearly misunderstanding me, I meant once you do your undergrad in a top 10 school and still don't get into a top 10 school for grad school, it gives you the feeling that you have not done as much as you should have done, it makes you feel that you had the opportunities but failed to utilize them, it makes you wonder whether you even deserved to be in the top 10 school you graduated from, and this self-doubt (a more intense version of impostor syndrome) causes the inferiority complex (you don't feel inferior to others, you feel inferior to what you are supposed to be).

FYI, I don't believe those who go to a top 10 schools are in any way superior to those who don't, I just believe going to a top 10 school gives you more opportunities, thats all. The number (not necessarily the quality) of 'great advisers' are much higher in top 10 schools (or so I think), so you have much more options, that is what sets them apart from other schools, not some stupid rank.
Last edited by jigsaw on Thu Mar 21, 2013 5:14 am, edited 1 time in total.

bfollinprm
Posts: 1203
Joined: Sat Nov 07, 2009 11:44 am

Re: Unexpectedly rejected almost everywhere, Please explain..

Post by bfollinprm » Thu Mar 21, 2013 5:08 am

jigsaw wrote:
blighter wrote:I don't think there is anything to be elaborated on here. If you feel inferior to the people from top 10, you probably also feel superior to the people outside of top 30. Also I believe you feel inferior because of your ego, as if you are entitled to a top 10 school. People have done well everywhere. I don't think you need to focus on the rankings as long as you get a decent adviser in your field of interest.
You are clearly misunderstanding me, I meant once you do your undergrad in a top 10 school and still don't get into a top 10 school for grad school, it gives you the feeling that you have not done as much as you should have done, it makes you feel that you got opportunities but failed to fully utilize them, that in turn makes you doubt whether you even deserved to be in the top 10 school you graduated from, and this self-doubt causes the inferiority complex. FYI, I don't believe those who go to a top 10 schools are in any way superior to those who don't, I just believe going to a top 10 school gives you more opportunities, thats all. The number (not necessarily the quality) of 'great advisers' are much higher in top 10 schools (or so I think), so you have much more options, that is what sets them apart from other schools, not some stupid rank.
MIT isn't the only place in the world that prepares you well for a PhD in physics.

jigsaw
Posts: 40
Joined: Sun Mar 17, 2013 10:59 am

Re: Unexpectedly rejected almost everywhere, Please explain..

Post by jigsaw » Thu Mar 21, 2013 5:19 am

bfollinprm wrote:MIT isn't the only place in the world that prepares you well for a PhD in physics.
Sure it isn't the 'only place', but it certainly is 'one of the places'. Lots of rejections makes you wonder whether you deserved to be part of 'one of the places'. Enough of psychology, lets get back to the business. Can you please take a look at the 5 points I made above, and let me know your opinion.
Last edited by jigsaw on Thu Mar 21, 2013 5:22 am, edited 1 time in total.

Godot
Posts: 12
Joined: Wed Feb 13, 2013 1:09 pm

Re: Unexpectedly rejected almost everywhere, Please explain..

Post by Godot » Thu Mar 21, 2013 5:20 am

jigsaw wrote:Now, I would like to add a few points of my own since I would like to know your detailed opinion about them :

(1) Flexible Option of Physics Major : This option is generally meant for students who aren't sure whether they want to do a graduate study in Physics or in some other field. Although my reasons of doing it was different (already explained), could the admission committee have frowned at the fact that I didn't do the 'Focused Option'? The only major differences are Senior Thesis, and Experimental Physics II. For whatever the reason may be and however well documented it may be, the end result is that I didn't have a Senior Thesis. How severe, if any, an effect can this have on graduate school admission? Unfortunately there is nothing I can do about it now, but still it would be good to know.
Having a thesis and a second semester of lab might have helped you, but not having them was probably not a major cause of your rejections. I got into top 5 programs without either of these things.
(4) Limited Math Coursework : I didn't take that many math classes at MIT, especially after my freshman year. I took 2 terms of Calculus, 1 term of Differential Equations, 1 term of Linear Algebra - the theoretical sequence (18.014, 18.024, 18.034, 18.700), not the standard sequence (18.01, 18.02, 18.03, 18.06). Beyond that I learned Math mostly in the context of Physics courses (as much as I needed, and whenever I needed) instead of taking separate Math courses. How severe, if any, an effect can this have on graduate school admission (especially in hep-th which is a little more mathematical than other fields)? Do you think taking GRE Math Subject test would have helped in graduate admission in hep-th? I took GR9768 all of a sudden yesterday, and although I made quite a few silly mistakes, I did manage 88 percentile; with a little practice (taking a few mock tests) I can easily make it 95 percentile (although a 99% will require a little more than that). If I have 95% in Math GRE as well, will it improve my chances if I indeed decide to re-apply (I've yet to decide whether I'll reapply or not)?
There is no reason to take the math GRE too; you got a 990 on the physics GRE, I think you're already maxed out in this area.
(5) Person of Interest in SOP : In my SOP for every school, I quite specifically mentioned which two professors of their school I'm most interested in working with. But I was worried that I may be rejected if those particular professors had no vacancy this year, so to be on the safe side, I wrote down the names of other professors I may be interested in as well in my SOP - this list was somewhat long, 5-10 professors depending on the school. I initially thought this was a good idea, but I'm not so sure anymore after the feedback from UT Austin. What effect, if any, (and how severe) can this have on graduate admission in your opinion?
Yes, this is bad. Listing 5-10 professors shows a lack of focus. It's okay to list like three professors to avoid pidgeonholing yourself and to show that you're a bit flexible, but listing tons of professors basically says "I don't actually have any idea about what I want to do"

I'd be willing to bet that by far your biggest mistake was using course recs instead of research recs. Your application is supposed to show the committee that you have the capability or potential to perform great research; the best way to convince them is to have already done that.

Finally, there are lots of great advisers anywhere. I had great experiences doing research for professors at a local, unranked college over the past several summers, and even got a PR B publication out of it (in CMT). The biggest difference between schools as you go up and down the ranks is likely to be the students.

friedrice
Posts: 8
Joined: Fri Dec 28, 2012 3:55 pm

Re: Unexpectedly rejected almost everywhere, Please explain..

Post by friedrice » Thu Mar 21, 2013 9:22 am

jigsaw wrote:
actrask wrote:Wait so you have got into Rutgers?

I don't think you mentioned this in your OP. Rutgers, although maybe not topping the rankings, is still a very good school for hep-th. It's underrated in my opinion.

Although you may think less of it since it was last on your list of "safety" schools, I don't think you should have that many reasons to count it out. You don't really have safety schools for hep-th anyway and you applied to almost all top-10 programs.
Yes I added some new information recently, so you may want to take a look at my posts once again. You may remember I wrote something like 'I have a place to go this time, it is a top 30 school, although not top 20' earlier, it was Rutgers I was talking about. I know it is a very decent school for hep-th (or else why would I apply for it), it is just people frown at you when you go to a school not in top 10 (or atleast top 20), and there is inferiority complex associated with going to a school that isn't ranked very high - perhaps you Americans don't feel that way, but this is the mindset of almost all Asians (including me).
Does anyone know whether it's possible to transfer between graduate schools? Can jigsaw do so if he decides he wants to try for a higher-ranked school next year? I'm kinda curious about this as well. I've heard of someone who stopped doing his PhD after about 1 year or so, he went to work, and then he decided to finish his PhD at another school. Both schools are very highly ranked...

crw888
Posts: 5
Joined: Tue Jan 01, 2013 4:20 pm

Re: Unexpectedly rejected almost everywhere, Please explain..

Post by crw888 » Thu Mar 21, 2013 10:08 am

friedrice wrote: Does anyone know whether it's possible to transfer between graduate schools? Can jigsaw do so if he decides he wants to try for a higher-ranked school next year? I'm kinda curious about this as well. I've heard of someone who stopped doing his PhD after about 1 year or so, he went to work, and then he decided to finish his PhD at another school. Both schools are very highly ranked...
That's what I'm doing right now, but I don't know if it's universal. This fall, I started grad school at the same school where I did my undergrad, and now I'm looking to transfer to another school to complete my master's. Three of the schools I've been in contact with have said that some, or all, of the coursework I've completed will transfer. I'm not working on a thesis at the moment, instead working on getting a paper written before I leave. One of the professors who made me an offer has suggested that I continue working on this project and turn it into a thesis. Between this, and the coursework I've completed already, I could potentially finish my master's in a year.

nerv
Posts: 7
Joined: Fri Feb 22, 2013 5:36 pm

Re: Unexpectedly rejected almost everywhere, Please explain..

Post by nerv » Thu Mar 21, 2013 11:23 am

My subjective views:
jigsaw wrote: (1) Flexible Option of Physics Major : This option is generally meant for students who aren't sure whether they want to do a graduate study in Physics or in some other field. Although my reasons of doing it was different (already explained), could the admission committee have frowned at the fact that I didn't do the 'Focused Option'? The only major differences are Senior Thesis, and Experimental Physics II. For whatever the reason may be and however well documented it may be, the end result is that I didn't have a Senior Thesis. How severe, if any, an effect can this have on graduate school admission? Unfortunately there is nothing I can do about it now, but still it would be good to know.
Doesn't seem to me like the flexible option is very different from the focused option. I would say that a senior thesis is not compulsory, but it is one way out of many that showcases your research / research abilities, which is important. So:

having a thesis + other research projects = great
having a thesis = good,
having no thesis + other research projects = equally good
having no thesis + no research projects = bad. you can't 'cruise' and get into a PhD program with classes alone.
jigsaw wrote: (2) Taking a Year Off : As stated earlier, I couldn't afford to study Part III of Math Tripos at University of Cambridge or MSc in Quantum Fields and Fundamental Forces at Imperial College London despite being offered admission (due to lack of financial aid), and had to take a year off. Although I did quite a bit of self study, as you know it is not something that can be 'documented well'. How severe, if any, an effect can this have on graduate school admission? Unfortunately there is nothing I can do about it now, but still it would be good to know.
A year off is fine. It's okay as long as you explain in your SOP persuasively why you still want to do a PhD after that time. Note that I didn't say 'explain what physics related things you've been doing in that year' because people take time off for a variety of reasons, usually non-academic related, and it would be silly to insist that every amount of time spent outside must be on physics in order for you to secure a place in a program in the future.

In fact, one of the profs in my school took 2, or 3 years off after undergraduate school. He worked as a carpenter, painter, doing odd jobs etc. to build a yacht. He sailed around the world for a bit, then decided to apply for graduate school.
jigsaw wrote: (3) Health Issues : I missed lots of classes at MIT at various points due to illness. Although I didn't mention it in my SOP (except for the senior year), I don't know if my recommenders did, in any case I had made a full recovery by the time I applied for grad school this year, and I mentioned that clearly in my SOP. Do you think it might have a negative influence on my graduate application if the schools somehow found out about it? If yes, how severe can the effect be?
No I don't think it matters much.
jigsaw wrote: (4) Limited Math Coursework : I didn't take that many math classes at MIT, especially after my freshman year. I took 2 terms of Calculus, 1 term of Differential Equations, 1 term of Linear Algebra - the theoretical sequence (18.014, 18.024, 18.034, 18.700), not the standard sequence (18.01, 18.02, 18.03, 18.06). Beyond that I learned Math mostly in the context of Physics courses (as much as I needed, and whenever I needed) instead of taking separate Math courses. How severe, if any, an effect can this have on graduate school admission (especially in hep-th which is a little more mathematical than other fields)? Do you think taking GRE Math Subject test would have helped in graduate admission in hep-th? I took GR9768 all of a sudden yesterday, and although I made quite a few silly mistakes, I did manage 88 percentile; with a little practice (taking a few mock tests) I can easily make it 95 percentile (although a 99% will require a little more than that). If I have 95% in Math GRE as well, will it improve my chances if I indeed decide to re-apply (I've yet to decide whether I'll reapply or not)?
Like the point I made about the thesis, math courses aren't compulsory, but it's good to have them. It 'shows' that one is able to think abstractly (which is necessary for theory in general), rather than just being able to solve textbook questions. However, I would like to put a disclaimer here that not all abstract math is useful for physics - like, I don't see how Galois theory might be useful (I might be wrong) to physics, or trying to formalize the Feynman path integral will not yield any physically interesting results... but the point is that at the very least taking, math classes trains one to think logically. can't say the same for physics where a lot of it is just plug (here's this formula!) and play (compute this).

about GREs - dude stop thinking about GREs. GREs are a stupid standardized form of testing that reveals nothing about a person's ability to do research. As long as your score is not below ~900 for hep-th you are fine. taking the math GRE is counterproductive, and silly at best.

in fact, dude - stop thinking about courses and exams and scores. all those are over right now and they will not help you at all now. Like, at all. the best way to help yourself now, if you want to apply again next year, is to find an adviser and work really hard on some project, and get some nice results you can call your own.
jigsaw wrote: (5) Person of Interest in SOP : In my SOP for every school, I quite specifically mentioned which two professors of their school I'm most interested in working with. But I was worried that I may be rejected if those particular professors had no vacancy this year, so to be on the safe side, I wrote down the names of other professors I may be interested in as well in my SOP - this list was somewhat long, 5-10 professors depending on the school. I initially thought this was a good idea, but I'm not so sure anymore after the feedback from UT Austin. What effect, if any, (and how severe) can this have on graduate admission in your opinion?
Listing 5-10 professors is like just copying the list from the department website. and wait, most schools have much fewer than 10 professors in hep-th. The norm is what, 3-4? so you basically listed the entire hep-th group? Do you see how that's a huge reg flag? Did you research on what each professor does and why you think you might want to work with him? (and did you give him a reason as to why he would want to work with you?)

Professors are humans too, and they have their ego. They want to feel like they're wanted by students, that they've been specially selected. Putting 5-10 of them down is like noncommittally saying to each one 'oh yea I guess I could work with you...' and no one likes to be just on some crap-shoot list.

TakeruK
Posts: 941
Joined: Mon Jan 02, 2012 3:05 pm

Re: Unexpectedly rejected almost everywhere, Please explain..

Post by TakeruK » Thu Mar 21, 2013 11:45 am

I'll give my response to your questions too!
jigsaw wrote:Now, I would like to add a few points of my own since I would like to know your detailed opinion about them :

(1) Flexible Option of Physics Major : This option is generally meant for students who aren't sure whether they want to do a graduate study in Physics or in some other field.
I don't think this matters. A person outside of MIT might not even know the distinction. Your description here is good but I hope you didn't take time in your SOP to explain this, since it's not important. In your shoes, I would also call the degree a "Physics Major" in my CV and any other material. Basically, I would not draw any attention to the distinction between Flexible and Focused options -- not because it matters, but because why make things more complicated. You are a Physics major -- that's all that matters. Even if your official transcript might denote your major as the "flexible option", I really doubt people will care as long as you have all of the standard BSc Physics coursework.
(2) Taking a Year Off :
Also doesn't matter. One year is nothing. This is something that might be worth 2 sentences in your SOP though -- you can say that after you graduated, you were accepted in these programs but could not afford to attend so you decided to do self-study instead. Don't dwell on it though. Usually I don't think people need to explain a 1-year gap between undergrad and applying to grad schools but in your case, it might be worth it because you can show positive aspects of yourself (i.e. not giving up, motivation to self-study).
(3) Health Issues : I missed lots of classes at MIT at various points due to illness. Although I didn't mention it in my SOP (except for the senior year), I don't know if my recommenders did, in any case I had made a full recovery by the time I applied for grad school this year, and I mentioned that clearly in my SOP. Do you think it might have a negative influence on my graduate application if the schools somehow found out about it? If yes, how severe can the effect be?
Probably not a big deal either. The admissions committee's job is NOT to decide whether or not you are healthy enough for graduate study -- they are not health professionals. They would likely trust your word on it -- i.e. if you didn't feel healthy enough for graduate work, then you would not have applied. If you really feel like you need to mention it to address some strange stuff in your application, you can briefly refer to some health issues you had. No details or medical letters needed. But unless you failed courses/semesters or missed semesters, I wouldn't even mention.
(4) Limited Math Coursework : I didn't take that many math classes at MIT, especially after my freshman year. I took 2 terms of Calculus, 1 term of Differential Equations, 1 term of Linear Algebra - the theoretical sequence (18.014, 18.024, 18.034, 18.700), not the standard sequence (18.01, 18.02, 18.03, 18.06).
Not having advanced math courses might not look great for someone entering such a math heavy field. But I don't think this is the major problem either. I wouldn't worry about the different sequences, again someone outside of MIT won't know or really care. They just want to know you have the right foundation.
Do you think taking GRE Math Subject test would have helped in graduate admission in hep-th?
No, I think you have enough "on-paper" credentials (e.g. major GPA and Physics GRE scores).
(5) Person of Interest in SOP : In my SOP for every school, I quite specifically mentioned which two professors of their school I'm most interested in working with. But I was worried that I may be rejected if those particular professors had no vacancy this year, so to be on the safe side, I wrote down the names of other professors I may be interested in as well in my SOP - this list was somewhat long, 5-10 professors depending on the school. I initially thought this was a good idea, but I'm not so sure anymore after the feedback from UT Austin. What effect, if any, (and how severe) can this have on graduate admission in your opinion?
I think this, and your LORs are the major problem areas in your application. 5-10 is way too many. 2 is a good number, and 4 is probably the maximum. It's the quality of the fit that counts, not the quantity of profs that fit! Did you contact these profs before your application? I know some people have different opinions about this, but I think it's a good idea to contact profs you want to work with about a month before applications are due. Not because you want to suck up to them and get them to help you be admitted, but it's important to know that their interests actually match yours and that they are able to take on students!

Also, about the whole inferiority complex thing -- in addition to what others said (i.e. if you think a top 10 school is so important, then you might subconsciously think research from lower ranked schools are less important and that's not a healthy attitude to have), I think having more confidence in yourself is important to success. Don't let the judgements of others (i.e. "oh, you're not in a Top 10 school?) make you feel inadequate. For example, in graduate school, you will have to find a balance between getting good grades and outputting good research. There's just not enough time to do both, unless you want to work 80 hours per week. One key skill in grad school, in my opinion, is to learn when to stop working on courses. School rankings, and course grades, are not a good measure of how successful you are. If you internalize it and believe that you're not good enough because of things like rankings or grades, then you might subconsciously pass it on to others you meet, and most people don't like meeting attitudes like that! I know it's easier said than done though. I also understand what you mean but it's important to move past that!

friedrice
Posts: 8
Joined: Fri Dec 28, 2012 3:55 pm

Re: Unexpectedly rejected almost everywhere, Please explain..

Post by friedrice » Thu Mar 21, 2013 11:54 am

crw888 wrote:
friedrice wrote: Does anyone know whether it's possible to transfer between graduate schools? Can jigsaw do so if he decides he wants to try for a higher-ranked school next year? I'm kinda curious about this as well. I've heard of someone who stopped doing his PhD after about 1 year or so, he went to work, and then he decided to finish his PhD at another school. Both schools are very highly ranked...
That's what I'm doing right now, but I don't know if it's universal. This fall, I started grad school at the same school where I did my undergrad, and now I'm looking to transfer to another school to complete my master's. Three of the schools I've been in contact with have said that some, or all, of the coursework I've completed will transfer. I'm not working on a thesis at the moment, instead working on getting a paper written before I leave. One of the professors who made me an offer has suggested that I continue working on this project and turn it into a thesis. Between this, and the coursework I've completed already, I could potentially finish my master's in a year.
That sounds great!

User avatar
Andromeda
Posts: 127
Joined: Wed Dec 10, 2008 3:17 pm

Re: Unexpectedly rejected almost everywhere, Please explain..

Post by Andromeda » Thu Mar 21, 2013 12:46 pm

Wow, I wasn't actually expecting the OP to repost everything. Thank you for doing so!

Regarding the transfer aspect, it is possible but I don't think anyone should consider it going into grad school as an option (short of taking an M.Sc. and reapplying with that). The reason I say this is the only people I've known who did it successfully were those who realized their research interests were better aligned with faculty at other universities, got in contact with professors at other places who then agreed to take them on, and got accepted through that. And that is within physics of course- I myself have done the transfer thing but it was physics to astronomy, so that's a slight bit different but as that's not what you want to do I will spare you the details.

So if you want to do a certain kind of theory which exists at the university you're going to, and then decide to transfer just because the other school's reputation is better, everyone will see through your ruse pretty quickly and I don't think they'll be very impressed. I will agree with others though in that Rutgers is a perfectly fine institution and department which will surely challenge you well so I wouldn't worry about this.

Put it this way, they have done studies at the undergraduate and graduate levels where people who are accepted to top-tier institutions but then can't attend and have to opt for a good but less prestigious institution (ie due to family, funding, etc) still end up being just as successful later in life. So if you really are that awesome and it was just your letters were awful or whatever you should still be able to do brilliant research and have a great career ahead of you.

If you really want to spend your life worrying about prestige, well you have a bit more learning to do of a different sort in my opinion.

jigsaw
Posts: 40
Joined: Sun Mar 17, 2013 10:59 am

Re: Unexpectedly rejected almost everywhere, Please explain..

Post by jigsaw » Thu Mar 21, 2013 1:09 pm

TakeruK wrote:Also, about the whole inferiority complex thing -- in addition to what others said (i.e. if you think a top 10 school is so important, then you might subconsciously think research from lower ranked schools are less important and that's not a healthy attitude to have), I think having more confidence in yourself is important to success. Don't let the judgements of others (i.e. "oh, you're not in a Top 10 school?) make you feel inadequate. For example, in graduate school, you will have to find a balance between getting good grades and outputting good research. There's just not enough time to do both, unless you want to work 80 hours per week. One key skill in grad school, in my opinion, is to learn when to stop working on courses. School rankings, and course grades, are not a good measure of how successful you are. If you internalize it and believe that you're not good enough because of things like rankings or grades, then you might subconsciously pass it on to others you meet, and most people don't like meeting attitudes like that! I know it's easier said than done though. I also understand what you mean but it's important to move past that!
Actually we are brought up in a totally different environment. In my home country, grades and test scores are considered everything - in fact college admission is based on standardized test scores only (no recommendation, essays etc), and even grad school admission is entirely based on standardized test scores + interviews (no recommendation, research etc); and it takes time to overcome the mindset we have acquired since childhood. My 4 years of college in US were absolutely refreshing, and now I'm much less obsessed with grades and test scores and ranks than I was 5 years ago, but I guess it will still take years for me to overcome it completely. Your comment was really helpful, thanks a lot. I'll try my best.

jigsaw
Posts: 40
Joined: Sun Mar 17, 2013 10:59 am

Re: Unexpectedly rejected almost everywhere, Please explain..

Post by jigsaw » Thu Mar 21, 2013 1:39 pm

OK, so in short, the general consensus is that the first 4 points I mentioned aren't very harmful, but the 5th point (POI in SOP) is. Basically I included two separate lists in my SOP - first, a list of Professors I'm truly interested in working with (2 professors for each school); and second, a separate list of all Professors I have even the slightest interest in (another 5-10 professors for each school). I thought it made my application safe, but everyone here thinks that it sends the wrong message that 'I am not sure about what I want to do in grad school' - which was totally unintentional. I should have optimally limited myself to 3-4 professors (in total) for each school. That makes sense now.

Now, about contacting professors before applying for grad school, are you sure that it is advisable? I certainly didn't contact anyone before I had an offer of acceptance. I was afraid that in most cases the professor will not bother to read (since he wouldn't know me); and even if he does read, he may get the wrong impression that I'm one of those annoying type of students who wastes professors' time even before they know whether they have been accepted at that grad school or not. Was I just being paranoid?

One more question, how important is it to explain your background, hardships and the obstacles you had to overcome, how you became interested in Physics, your achievements and failure in high school (i.e. before college) - this sort of things in the SOP? In the CV, should one include the achievements etc. in high school, or should one just confine himself to the years in college? Similarly, should one include high school transcripts along with college transcripts? Does these things matter for grad school admission? I wasn't sure, so in most cases I included everything that was important to me (not extra curricular though).

blighter
Posts: 256
Joined: Thu Jan 26, 2012 6:30 pm

Re: Unexpectedly rejected almost everywhere, Please explain..

Post by blighter » Thu Mar 21, 2013 2:07 pm

jigsaw wrote:
TakeruK wrote:Also, about the whole inferiority complex thing -- in addition to what others said (i.e. if you think a top 10 school is so important, then you might subconsciously think research from lower ranked schools are less important and that's not a healthy attitude to have), I think having more confidence in yourself is important to success. Don't let the judgements of others (i.e. "oh, you're not in a Top 10 school?) make you feel inadequate. For example, in graduate school, you will have to find a balance between getting good grades and outputting good research. There's just not enough time to do both, unless you want to work 80 hours per week. One key skill in grad school, in my opinion, is to learn when to stop working on courses. School rankings, and course grades, are not a good measure of how successful you are. If you internalize it and believe that you're not good enough because of things like rankings or grades, then you might subconsciously pass it on to others you meet, and most people don't like meeting attitudes like that! I know it's easier said than done though. I also understand what you mean but it's important to move past that!
Actually we are brought up in a totally different environment. In my home country, grades and test scores are considered everything - in fact college admission is based on standardized test scores only (no recommendation, essays etc), and even grad school admission is entirely based on standardized test scores + interviews (no recommendation, research etc); and it takes time to overcome the mindset we have acquired since childhood. My 4 years of college in US were absolutely refreshing, and now I'm much less obsessed with grades and test scores and ranks than I was 5 years ago, but I guess it will still take years for me to overcome it completely. Your comment was really helpful, thanks a lot. I'll try my best.
Are you from India? If that's the case count yourself lucky that you went to US. I didn't have enough foresight to see through the flaws in the system back then. The entire system is horrible. If you think it's just the admissions, you'd be wrong. Even all the classes in college are like that. You are graded just based on a couple of exams. The exams are a complete joke. We get three hours and a bunch of questions which cannot be solved in three hours. The people who solve the most get A's and people like me who are slightly slower end up getting D's and even F's. The workload is around 60 hours a week. It's all about competition all the way. I think the problem lies in the fact that those who end up being professors here have gone through this very same system and they happen to be the lucky ones who actually flourished in this. So the support for the system gets reinforced. I believe that's the reason you don't see quality research in India. Nobody really cares about research. It's all a rat race chasing grades.

Kinbote
Posts: 24
Joined: Tue Feb 12, 2013 7:30 pm

Re: Unexpectedly rejected almost everywhere, Please explain..

Post by Kinbote » Thu Mar 21, 2013 3:52 pm

Until now everyone seems to agree that not having a strong math background isn't really an issue.
I disagree - if you want to do theory, condensed matter or high energy, you're going to have to have a fantastic math background. By this I mean, rigorous courses in group theory, differential geometry (which is essential in string theory), functional analysis etc. PDEs and Linear ALgebra is a given.
I understand the argument that often these subjects are covered in physics courses themselves and that the mathematical level of rigour is unnecessary for physicists but having that background gives you a solid foundation in logic and writing proofs - even if you're doing phenomenology, you're going to have to string together certain pieces of evidence in a particular order to get at your conclusion, which is precisely what math courses teach you to do, albeit very rigourously.

And I second everything that's been said about rankings et al.
For instance, I got waitlisted at Harvard and they said there might be a slight chance I could attend, but there is only one professor I want to work with, who is very famous and doesn't really work closely with his graduate students. So the chances that I'll actually get to work with him, rather than just in his group, are extremely slim. Contrary to that, there's three profs at UPenn and Boulder and UCSB, each of whom is doing precisely what I want to do and there's a much greater chance I'll get to actually work with them. So despite Harvard being the obvious choice in terms of rankings, I doubt I'd be able to do the level of research I might do at these other places.



Post Reply